Đến nội dung

Nguyenhuyen_AG nội dung

Có 785 mục bởi Nguyenhuyen_AG (Tìm giới hạn từ 29-04-2020)



Sắp theo                Sắp xếp  

#483805 Bạn đã tìm lời giải như thế nào ?

Đã gửi bởi Nguyenhuyen_AG on 18-02-2014 - 11:52 trong Tài liệu, chuyên đề, phương pháp về Bất đẳng thức

Tình cờ thấy được bài này trên diễn đàn mọi người cùng phân tích thử xem nhé. 

 

$\boxed{\text{Bài 6.}}$ Cho hai số dương $a,\;b$ thỏa mãn điều kiện $a^2+b^2=5.$ Hãy tìm giá trị nhỏ nhất của biểu thức

\[P=a^3+b^6.\]




#485124 Bạn đã tìm lời giải như thế nào ?

Đã gửi bởi Nguyenhuyen_AG on 28-02-2014 - 15:14 trong Tài liệu, chuyên đề, phương pháp về Bất đẳng thức

Tình cờ thấy được bài này trên diễn đàn mọi người cùng phân tích thử xem nhé. 

 

$\boxed{\text{Bài 6.}}$ Cho hai số dương $a,\;b$ thỏa mãn điều kiện $a^2+b^2=5.$ Hãy tìm giá trị nhỏ nhất của biểu thức

\[P=a^3+b^6.\]

 

Bài này có ý tưởng AM-GM rất rõ ràng, giả thiết của bài toán chứa biểu thức có bậc nhỏ hơn bậc của $P,$ và việc sử dụng bất đẳng thức AM-GM để hạ bậc là một ý tưởng rất tự nhiên. Tuy nhiên, chúng ta lại không biết là dấu bằng của bài toán sẽ xảy ra khi nào nên việc chọn hệ số để sử dụng bất đẳng thức AM-GM cũng khá vất vả, vì vậy ta sẽ sử dụng kỹ thuật chọn dấu bằng giả định.

 

Cụ thể, ta sẽ giả sử đẳng thức của bài toán sẽ đạt được tại $a=x,\;b=y$ với $x,\;y$ là những hằng số. Khi đó theo bất đẳng thức AM-GM, ta có

\[a^3+a^3+x^3 \ge 3\sqrt[3]{a^3\cdot a^3 \cdot x^3}=3a^2\cdot x, \quad (1)\]

tức là

\[2a^3+x^3 \ge 3a^2\cdot x,\]

\[a^3\ge \frac{3a^2\cdot x-x^3 }{2}.\]

Hoàn toàn tườn tự, ta có 

\[b^6\ge 3b^2\cdot y^4-2y^6\]

Như vậy, ta có 

\[ a^3+b^6 \ge 3b^2\cdot y^4-2y^6+\frac{3a^2\cdot x}{2}-\frac{x^3}{2}=3\left ( b^2\cdot y^2+a^2\cdot \frac{x}{2} \right )-2y^6-\frac{x^3}{2}. \quad (2)\]

Ta cần chọn $x,\;y$ sao cho trong $(2)$ ta có thể sử sụng được giả thiết $a^2+b^2=5$ đồng thời cũng thoả mãn điều kiện của bait tóan, tức $x,\;y$ phải thoả mãn hệ  phương trình

\[\left\{\begin{matrix}x^2+y^2&=5 \\y^2&=\frac{x}{2}\end{matrix}\right.\]

Giải hệ này ta được $x=2,\;y=1$ và suy ra $P_{\min} =9.$

 

Nhận xét: Có một số bạn sẽ đặt câu hỏi rằng tại sao lại dùng $2$ đại lượng $a^3$ kết hợp với $x^3$ để sử dụng bất đẳng thức AM-GM cho ba số. Để ý rằng điều kiện của bài toán cho ta $a^2+b^2=5$ nên chúng ta phải cố gắng đánh giá $a^3$ sao cho xuất hiện được $a^2$ và $b^6$ sao cho xuất hiện được $b^2$ để áp dụng giả thiết $a^2+b^2=5.$ Điều này lý giải cho việc áp dụng bất đẳng thức AM-GM như trên.




#485266 Bạn đã tìm lời giải như thế nào ?

Đã gửi bởi Nguyenhuyen_AG on 01-03-2014 - 11:52 trong Tài liệu, chuyên đề, phương pháp về Bất đẳng thức


 Tương tự ta có bài toán sau:
Cho $a,b,c$ thực dương thỏa $2a+2b+c^2=14$. Tìm giá trị nhỏ nhất của $$P=a^3+2b^2+2c^3$$ (Trích đề thi thử Moon.vn lần 2-2014)
 

Spoiler

 

Hoàn toàn tương tự. Ta giả sử $a=x,\;b=y,\;c=z$ và sử dụng bất đẳng thức AM-GM để tạo ra $a,\;b$ và $b^2.$ Ta có

\[a^3+2b^2+2c^3 \ge \left ( 3ax^2+4by+3c^2z \right )-\left ( 2x^3+2y^2+z^3 \right ).\]

Để sử dụng được điều kiện thì ta cần tách

\[3ax^2+4by+3c^2z=2a\cdot \frac{3x^2}{2}+2b\cdot2y+c^2\cdot3z.\]

Như vậy ta cần chọn $x,\;y,\;z$ thoả mãn điều kiện

\[\left\{\begin{matrix}&\dfrac{3x^2}{2}=2y=3z \\& 2x+2y+z^2=14\end{matrix}\right.\] 

Giải hệ này ta được $x=2,\;y=3,\;z=2$ và có được $P_{\min} = 34.$




#489199 Bạn đã tìm lời giải như thế nào ?

Đã gửi bởi Nguyenhuyen_AG on 28-03-2014 - 16:19 trong Tài liệu, chuyên đề, phương pháp về Bất đẳng thức

Mọi người tiếp tục nhé. Đây là bài thi quốc gia năm 2008.

 

$\boxed{\text{Bài 7.}}$ Với $x,\,y,\,z$ là ba số thực không âm đôi một khác nhau, hãy chứng minh

\[(xy+yz+zx)\left [\frac{1}{(x-y)^2}+\frac{1}{(y-z)^2}+\frac{1}{(z-x)^2} \right ] \ge 4.\]




#508579 $\frac{a+b+c}{3}-\sqrt[3]{abc}...

Đã gửi bởi Nguyenhuyen_AG on 23-06-2014 - 14:15 trong Bất đẳng thức - Cực trị

Cho a,b,c là các số thực dương .CMR : 

 

      $$\frac{a+b+c}{3}-\sqrt[3]{abc}\leq max\left \{ (\sqrt{a}-\sqrt{b})^2,(\sqrt{b}-\sqrt{c})^2,(\sqrt{c}-\sqrt{a})^2 \right.\left. \right \}$$

Hãy tìm hằng số $k$ tốt nhất của bất đẳng thức

\[\frac{a+b+c}{3}-\sqrt[3]{abc}\leq k\cdot\max\left \{ (\sqrt{a}-\sqrt{b})^2,\,(\sqrt{b}-\sqrt{c})^2,\,(\sqrt{c}-\sqrt{a})^2\right\},\]

với $a,\,b,\,c$ là các số thực không âm.




#519313 $\frac{1}{a+b}+\frac{1}{b+c...

Đã gửi bởi Nguyenhuyen_AG on 13-08-2014 - 17:47 trong Bất đẳng thức - Cực trị

Cho $a,b,c\geq 0$ thỏa mãn $ab+bc+ca=1$. CMR:
$$\frac{1}{a+b}+\frac{1}{b+c}+\frac{1}{c+a}-\frac{1}{a+b+c}\geq 2$$

Bất đẳng thức cần chứng minh tương đương với

$$\frac{1}{a+b}+\frac{1}{b+c}+\frac{1}{c+a}\geq 2+\frac{1}{a+b+c},$$

$$\frac{a+b+c}{a+b}+\frac{a+b+c}{b+c}+\frac{a+b+c}{c+a}\geq 2(a+b+c)+1,$$

$$\frac{c}{a+b}+\frac{a}{b+c}+\frac{b}{c+a}+2\geq 2(a+b+c).$$

Áp dụng bất đẳng thức Cauchy-Schwarz và AM-GM ta có

$$\frac{c}{a+b}+\frac{a}{b+c}+\frac{b}{c+a}+2\ge \frac{(a+b+c)^2}{2(ab+bc+ca)}+2=\frac{(a+b+c)^2}{2}+2 \geq 2(a+b+c).$$

Đẳng thức xảy ra khi $a=b,c=0$ cùng các hoán vị.




#519314 CMR:$\frac{ab}{3a^2+b^2}+\frac{bc...

Đã gửi bởi Nguyenhuyen_AG on 13-08-2014 - 17:49 trong Bất đẳng thức - Cực trị

Gợi ý: Tách $3a^2+b^2=2a^2+(a^2+b^2)$ rồi sử dụng bất đẳng thức AM-GM.




#519336 CMR:$\frac{ab}{3a^2+b^2}+\frac{bc...

Đã gửi bởi Nguyenhuyen_AG on 13-08-2014 - 19:47 trong Bất đẳng thức - Cực trị

Gợi ý: Tách $3a^2+b^2=2a^2+(a^2+b^2)$ rồi sử dụng bất đẳng thức AM-GM.

 

Bạn hãy đặt bút suy nghĩ thêm tí nữa, chưa gì đã bảo không được rồi.




#520782 $\frac{1}{a+b}+\frac{1}{b+c...

Đã gửi bởi Nguyenhuyen_AG on 22-08-2014 - 21:01 trong Bất đẳng thức - Cực trị

Đoạn $$\sum \frac{a}{b+c}\geq \frac{(\sum a)^2}{2\sum ab}$$

Thì dấu $=$ phải ở $a=b=c$ chứ anh nhỉ ?

Em xem kỹ lại, đánh giá này có 2 dấu bằng.




#522689 $$(a^3+b)(b^3+c)(c^3+a)+10\leq 6(a^2+b^2+c^2)$$

Đã gửi bởi Nguyenhuyen_AG on 04-09-2014 - 08:47 trong Bất đẳng thức - Cực trị

Bài toán .
Ch0 các số thực dương $a,b,c$ có tổng bằng 3.Chứng minh rằng:
$$(a^3+b)(b^3+c)(c^3+a)+10\leq 6(a^2+b^2+c^2)$$

Bất đẳng thức này không đúng.




#525071 $kho - tai - lieu - bat - dang - thuc$

Đã gửi bởi Nguyenhuyen_AG on 18-09-2014 - 13:58 trong Tài liệu, chuyên đề, phương pháp về Bất đẳng thức

Không biết bạn có tài liệu này không ? Tài liệu của diễn đàn batdangthuc.net, trong tài liệu có các bài toán Pro of day của diễn đàn batdangthuc.net, trong tài liẹu có ghi câu anh hùng hội tụ theo kiểu truyện kiếm hiệp.




#527058 $(a + b)(b + c)(c + a) \geq 2(a + b + c + 1)$

Đã gửi bởi Nguyenhuyen_AG on 03-10-2014 - 22:02 trong Bất đẳng thức và cực trị

Cho abc = 1

a) $(a + b)(b + c)(c + a) \geq 2(a + b + c + 1)$

b) $(a + b)(b + c)(c + a) \geq 4(a + b + c - 1)$

Câu a) chính là bài 3 trong đề thi APMO 1998, có một lời giải bằng Cauchy-Schwarz cho bài này. Còn câu b) là đề MOSP năm 2 nghìn lẻ mấy không nhớ. =)) Câu này có khá nhiều cách chứng minh. Xin trình bày một chứng minh bằng AM-GM.

 

Ta viết bất đẳng thức lại dưới dạng thuần nhất

\[\frac{(a + b)(b + c)(c + a)}{abc} \geqslant 4\left(\frac{a + b + c}{\sqrt[3]{abc}} - 1\right),\]

\[\frac{(a + b)(b + c)(c + a)}{abc} +4 \geqslant \frac{4(a + b + c)}{\sqrt[3]{abc}}.\]

Áp dụng bất đẳng thức AM-GM, ta có

\[\begin{aligned} \frac{(a + b)(b + c)(c + a)}{abc} +4 &=\frac{(a + b)(b + c)(c + a)}{2abc}+\frac{(a + b)(b + c)(c + a)}{2abc}+4 \\& \geqslant 3\sqrt[3]{\frac{(a + b)^2(b + c)^2(c + a)^2}{a^2b^2c^2}}.\end{aligned}\]

Vậy ta cần phải chứng minh

\[3\sqrt[3]{\frac{(a + b)^2(b + c)^2(c + a)^2}{a^2b^2c^2}} \geqslant \frac{4(a + b + c)}{\sqrt[3]{abc}},\]

lập phương 2 vế ta được

\[27(a + b)^2(b + c)^2(c + a)^2 \geqslant 64abc(a + b + c)^3.\]

Bất đẳng thức này đúng theo hai bất đẳng thức cơ bản sau:

\[(a+b)(b+c)(c+a) \ge \geqslant \frac{8(a+b+c)(ab+bc+ca)}{9},\]

\[(ab+bc+ca)^2 \ge 3abc(a+b+c).\]

Đẳng thức xảy ra khi $a=b=c.$ Bài toán được chứng minh




#527119 CMR: $2ab(a+b)+2bc(b+c)+2ca(c+a)\geq a^3+b^3+c^3+9abc$

Đã gửi bởi Nguyenhuyen_AG on 04-10-2014 - 15:30 trong Bất đẳng thức và cực trị

Đổi biến $a=x+y,\,b==y+z,\,c=z+x$ thì ta sẽ được bất đẳng thức Schur bậc 3.




#531773 $\sum \frac{a^{2}+b^{2}}{a+b}\leq 3\frac{a^{2}+b^{2}...

Đã gửi bởi Nguyenhuyen_AG on 04-11-2014 - 16:26 trong Bất đẳng thức và cực trị

Bất đẳng thức này đúng theo bất đẳng thức Schur và Cauchy-Schwarz.




#532400 [Chuẩn hóa] $\frac{(2a+b+c)^2}{2a^2+(b+c)^2}+...

Đã gửi bởi Nguyenhuyen_AG on 08-11-2014 - 20:28 trong Bất đẳng thức - Cực trị

còn việc chuẩn hóa là dành cho bđt đối xứng và thuần nhất

bđt đối xứng thì đã biết còn bđt thuần nhất là khi thay $a=tx,b=ty,c=tz$ thì bđt không thay đổi

ví dụ như CM $\frac{8(a^2+b^2+c^2)}{ab+bc+ca}+\frac{27(a+b)(b+c)(c+a)}{(a+b+c)^3}\geq 16$$(a,b,c>0)$

khi đặt $a=tx,b=ty,c=tz$ thì bđt vẫn không thay đổi

khi mới học cái chuẩn hóa này thì nghĩ tới câu hỏi sao lại có thể đặt $a+b+c=3,abc=1,...$ ta hiểu đơn giản như sau

nếu đặt $x=\frac{a}{a+b+c},y=\frac{b}{a+b+c},z=\frac{c}{a+b+c}\Rightarrow x+y+z=1$ và khi thay vào bđt ắt hẳn bđt sẽ không đổi

nếu đặt $x=\frac{a}{\sqrt[3]{abc}},y=\frac{b}{\sqrt[3]{abc}},z=\frac{c}{\sqrt[3]{abc}}\Rightarrow xyz=1$

và mấy cách chuẩn hóa khác cũng tương tự như vậy

 

NTP

Nhận định này của em sai nhé.




#532402 $\sum cos\frac{A-B}{2}\geq \sum...

Đã gửi bởi Nguyenhuyen_AG on 08-11-2014 - 20:32 trong Bất đẳng thức - Cực trị

   Cho $A,B,C$ là 3 góc của một tam giác. CMR :

 

      $cos\frac{A-B}{2}+cos\frac{B-C}{2}+cos\frac{C-A}{2}\geq sin\frac{3A}{2}+sin\frac{3B}{2}+sin\frac{3C}{2}$

 

 

 

 P/s: Bài này khá hay

USA TST 2003: http://www.artofprob...hp?f=54&t=3911




#532409 [Chuẩn hóa] $\frac{(2a+b+c)^2}{2a^2+(b+c)^2}+...

Đã gửi bởi Nguyenhuyen_AG on 08-11-2014 - 21:01 trong Bất đẳng thức - Cực trị

dạ,hình như thuần nhất thôi là được đúng không ạ,do em từng đọc tài liệu thì nó ghi là dành cho bđt đối xứng và hoán vị nhưng trong quá trình làm thì em thấy chỉ cần bđt thuần nhất thôi là được

 

NTP

Chỉ cần với bất đẳng thức thuần nhất là ta có thể chuẩn hóa được. Chuẩn hóa là một kỹ thuật khá quan trọng đặc biệt là với trong phương pháp U.C.T. Tuy nhiên có một số đã hiểu sai và to tát quá kỹ thuật này, thậm chí với số bạn rất giỏi bất đẳng thức nhưng không hiểu rõ được bản chất của chuẩn hóa. Ta có thể hiểu chuần hóa là phép đặt ẩn phụ.




#532421 $\sum a^{3}+\sum\frac{ab}{a^{2}+b^{2}}\geq \fra...

Đã gửi bởi Nguyenhuyen_AG on 08-11-2014 - 21:30 trong Bất đẳng thức và cực trị

 chứng minh bất đẳng thức sau với a,b,c>0 và abc=1

$a^{3}+b^{3}+c^{3}+\frac{ab}{a^{2}+b^{2}}+\frac{bc}{b^{2}+c^{2}}+\frac{ca}{c^{2}+a^{2}}\geq \frac{9}{2}$

 

Một lời giải bằng Cauchy-Schwarz. :icon6:  Ta viết bài toán về dạng thuần nhất như sau

\[\frac{a^{3}+b^{3}+c^{3}}{abc}+\frac{ab}{a^{2}+b^{2}}+\frac{bc}{b^{2}+c^{2}}+\frac{ca}{c^{2}+a^{2}}\geqslant \frac{9}{2},\]

tương đương với

\[\frac{2(a^{3}+b^{3}+c^{3})}{abc}+\frac{(a+b)^2}{a^{2}+b^{2}}+\frac{(b+c)^2}{b^{2}+c^{2}}+\frac{(c+a)^2}{c^{2}+a^{2}}\geqslant 12.\]

Theo bất đẳng thức Cauchy-Schwarz thì

\[\frac{(a+b)^2}{a^{2}+b^{2}}+\frac{(b+c)^2}{b^{2}+c^{2}}+\frac{(c+a)^2}{c^{2}+a^{2}}\geqslant \frac{\left (\displaystyle\sum a+b \right )^2}{\displaystyle\sum \left ( a^2+b^2 \right )}=\frac{2(a+b+c)^2}{a^2+b^2+c^2}=2+\frac{4(ab+bc+ca)}{a^2+b^2+c^2}.\]

Vậy ta cần chứng minh

\[\frac{a^3+b^3+c^3}{abc}+\frac{2(ab+bc+ca)}{a^2+b^2+c^2} \geqslant 5,\]

hay là

\[\frac{a^3+b^3+c^3}{abc}-3\geqslant 2\left (1-\frac{ab+bc+ca}{a^2+b^2+c^2}   \right ),\]

\[\frac{(a+b+c)(a^2+b^2+c^2-ab-bc-ca)}{abc}\geqslant \frac{2(a^2+b^2+c^2-ab-bc-ca)}{a^2+b^2+c^2},\]

\[(a+b+c)(a^2+b^2+c^2) \geqslant 2abc.\]

Nhưng ta luôn có

\[(a+b+c)(a^2+b^2+c^2) \geqslant 9abc > 2abc.\]

Ta có điều phải chứng minh.




#532463 Topic ôn luyện VMO 2015

Đã gửi bởi Nguyenhuyen_AG on 09-11-2014 - 00:40 trong Thi HSG Quốc gia và Quốc tế

Bài 16: Cho $A,B,C$ là 3 góc của một tam giác. CMR:

 

    $\cos\frac{A-B}{2}+\cos\frac{B-C}{2}+\cos\frac{C-A}{2}\geq \sin\frac{3A}{2}+\sin\frac{3B}{2}+\sin\frac{3C}{2}$.

 

Đây là đề thi TST của USA năm 2002, duới đây là một lời giải của Maverick

 

Let $p$ denote the semiperimeter of $\triangle ABC.$ WLOG assume $a \geq b \geq c.$
\[\cos\frac{A-B}2 =\cos\frac{A}2\cos\frac{B}2+\sin\frac{A}2\sin\frac{B}2 =  \sqrt{\frac{p^2(p-a)(p-b)}{abc^2}} +  \sqrt{\frac{(p-a)(p-b)(p-c)^2}{abc^2}}\]
\[\implies \sum \cos\frac{A-B}2= \sum \frac{a+b}c \sin\frac{C}2\]
Hence the inequality is equivalent to
\[\sum \frac{a+b}c\sin\frac{C}2 \geq 3\sum \sin\frac{A}2-4\sum \sin^3\frac{A}2\]
\[\iff \sum \frac{(b-c)(a-c)(a+b+c)}{abc} \sin\frac{C}2 \geq 0\]
\[\iff \sum (c-b)(c-a) \sin\frac{C}2 \geq 0\]
Due to assumption we have $(a-c)(b-c)\sin\frac{C}2 \geq 0$
So, it suffices to show that
\[(b-a)(c-a)\sin\frac{A}2+(a-b)(c-b)\sin\frac{B}2 \geq 0\]
\[\iff (a-b)\left((a-c)\sin\frac{A}2-(b-c)\sin\frac{B}2\right) \geq 0\]
which is obviously true.
 




#536329 Hãy phân tích số 89 thành tổng của các số nguyên dương sao cho tích của chúng...

Đã gửi bởi Nguyenhuyen_AG on 05-12-2014 - 20:24 trong Bất đẳng thức - Cực trị

Bài này có nguồn gốc từ IMO 1976 "http://www.artofprob...1715bc#p367432"




#566061 $\sum 2a^{2}b^{2} - \sum a^{4}...

Đã gửi bởi Nguyenhuyen_AG on 15-06-2015 - 22:53 trong Bất đẳng thức và cực trị

Cho $a,b,c$ là số đo ba cạnh của tam giác, chứng minh rằng

$\sum 2a^{2}b^{2} - \sum a^{4} > 0$

 

don't

Ta có

\[\sum 2a^{2}b^{2} - \sum a^{4} =(a+b+c)(a+b-c)(b+c-a)(c+a-b) \ge 0.\]




#566062 $(a+b+c)^2(a+b)(b+c)(c+a)\geq 24(a^2+b^2+c^2)abc$

Đã gửi bởi Nguyenhuyen_AG on 15-06-2015 - 23:03 trong Bất đẳng thức và cực trị

 Cho các số thực $a,b,c$ không âm. Chứng minh :

$$(a+b+c)^2(a+b)(b+c)(c+a)\geq 24(a^2+b^2+c^2)abc$$ 

 

 

Yêu cầu

 

Chuẩn hóa cho $a+b+c=3,$ và đặt $a^2+b^2+c^2=3+6t^2,\; (0\le t<1).$ Bất đẳng thức cần chứng minh được viết lại như sau
\[3\left(\dfrac{1}{a}+\dfrac{1}{b}+\dfrac{1}{c}\right) \ge 1+\dfrac{24(3+6t^2)}{9},\]
hay là

\[\dfrac{1}{a}+\dfrac{1}{b}+\dfrac{1}{c}\ge 3+\dfrac{16t^2}{3}.\]

Nếu $t=0$ thì $a=b=c=1,$ bất đẳng thức hiển nhiên đúng nên ta chi cần xét $0<t<1.$ Sử dụng đẳng thức

\[\dfrac{1}{a}=\dfrac{1}{1+2t}+\dfrac{1+2t-a}{a(1+2t)},\]

ta viết bất đẳng thức trên lại như sau

\[\displaystyle \frac{3}{1+2t}+\frac{1}{1+2t}\left(\dfrac{1+2t-a}{a}+\frac{1+2t-b}{b}+\frac{1+2t-c}{c}\right)\ge 3+\frac{16t^2}{3},\]

Dễ dàng chứng minh được rằng các số $\displaystyle 1+2t-a,1+2t-b,1+2t-c$ đều không âm. Từ đó, theo bất đẳng thức Cauchy-Schwarz, ta có

\[\begin{aligned}\dfrac{1+2t-a}{a}+\dfrac{1+2t-b}{b}+\dfrac{1+2t-c}{c}&\ge \dfrac{[(1+2t-a)+(1+2t-b)+(1+2t-c)]^2}{(1+2t-a)a+(1+2t-b)b+(1+2t-c)c}\\&=\dfrac{36t^2}{(1+2t)(a+b+c)-(a^2+b^2+c^2)}\\&=\dfrac{36t^2}{3(1+2t)-(3+6t^2)}\\&=\dfrac{6t}{1-t}.\end{aligned}\]

Vậy ta chỉ cần chứng minh được

\[\dfrac{3}{1+2t}+\dfrac{6}{(1+2t)(1-t)}\ge3+\dfrac{16t^2}{3},\]

hay là

\[\dfrac{6}{(1+2t)(1-t}-\dfrac{6t}{1+2t}\ge\dfrac{16t^2}{3},\]

\[\dfrac{6^2}{(1+2t)(1-t}\ge\dfrac{16t^2}{3},\]

\[8(1+2t)(1-t)\le9.\]
Sử dụng bất đẳng thức AM-GM, ta có \[8(1+2t)(1-t)=4\cdot (1+2t)\cdot 2(1-t)\le[(1+2t)+2(1-t)]^2=9.\]

Vậy ta có điều phải chứng minh, đẳng thức xảy ra khi và chỉ khi $a=b=c$ hoặc $a=2b=2c$ cùng các hoán vị tương ứng.




#566064 CMR:$\sum \sqrt{\frac{a}{b+c}...

Đã gửi bởi Nguyenhuyen_AG on 15-06-2015 - 23:08 trong Bất đẳng thức và cực trị

Cho $a,b,c\geq 0$ và không có đồng thời hai số bằng không. Chứng minh rằng $\sqrt{\frac{a}{b+c}}+\sqrt{\frac{b}{c+a}}+\sqrt{\frac{c}{a+b}}\geq 2\sqrt{1+\frac{abc}{(a+b)(b+c)(c+a)}}$

Spoiler

 

Chuẩn hoán $ab+bc+ca=1,$ với chú ý $a(a+b)(a+c) = a(a^2+ab+bc+ca) = a(a^2+1) = a^3+a.$ Bất đẳng thức trở thành

\[\sqrt{a^3+a}+\sqrt{b^3+b}+\sqrt{c^3+c}\ge2\sqrt{a+b+c}.\]

Áp dụng bất đẳng thức Minkowski, ta có

\[\begin{aligned} \sum \sqrt{a^3+a} &=\sqrt{a^2(a+b+c)+abc}+\sqrt{b^2(a+b+c)+abc}+\sqrt{c^2(a+b+c)+abc}\\&\ge\sqrt{\left(a\sqrt{a+b+c}+b\sqrt{a+b+c}+c\sqrt{a+b+c}\right)^2+\left(\sqrt{abc}+\sqrt{abc}+\sqrt{abc}\right)^2}\\&=\sqrt{(a+b+c)^3+9abc}\end{aligned}\]

Do đó ta cần chứng minh được

\[(a+b+c)^3+9abc\ge 4(a+b+c)(ab+bc+ca).\]

Đây chính là bất đẳng thức Schur bậc $3,$ đẳng thức xảy ra khi và chỉ khi $a=b=c $ hoặc $a=b,\;c=0$ và các hoán vị. Bài toán được chứng minh.




#566074 Topic tổng hợp một số bất đẳng thức trong kì thi MO các nước

Đã gửi bởi Nguyenhuyen_AG on 15-06-2015 - 23:49 trong Bất đẳng thức - Cực trị

  Bài 72 (IMO 2005): Cho các số thực dương $a,b,c$ thỏa mãn $abc\geq 1$. CMR:

 

      $\frac{a^5-a^2}{a^5+b^2+c^2}+\frac{b^5-b^2}{b^5+c^2+a^2}+\frac{c^5-c^2}{c^5+a^2+b^2}\geq 0$

 

Lời giải đặc biệt của kỳ thi.

 

Ta có

\[\frac{a^5-a^2}{a^5+b^2+c^2}-\frac{a^5-a^2}{a^3(a^2+b^2+c^2)}=\frac{(a-1)^2(a^2+a+1)(b^2+c^2)}{a(a^5+b^2+c^2)(a^2+b^2+c^2)} \ge 0.\]

Suy ra

\[\sum \frac{a^5-a^2}{a^5+b^2+c^2} \ge \sum\frac{a^5-a^2}{a^3(a^2+b^2+c^2)} = \sum \frac{a^2-\frac{1}{a}}{a^2+b^2+c^2} \ge \sum \frac{\sum a^2-\sum bc}{a^2+b^2+c^2} \ge 0.\]
---------------

@ducvipdh12: chào mừng sự trở lại của anh Huyện,lâu lắm mới thấy anh onl diễn đàn :))




#566269 $3(a^4+b^4+c^4)+a^2+b^2+c^2+6\geq 6(a^3+b^3+c^3)$

Đã gửi bởi Nguyenhuyen_AG on 16-06-2015 - 20:57 trong Bất đẳng thức - Cực trị

Bài toán: Cho $a,b,c$ là các số thực không âm thỏa $a+b+c=3$.Chứng minh rằng:

 

$3(a^4+b^4+c^4)+a^2+b^2+c^2+6\geq 6(a^3+b^3+c^3). \quad (2.2.1)$

 

Spoiler

 

Đặt $a=x+1,\,b=y+1,\,c=z+1,$ khi đó \[x+y+z= (a-1) + (b-1) + (c-1) = 0.\] Với phép đặt này thì \[\begin{aligned} a^2+b^2+c^2&=(x+1)^2+(y+1)^2+(z+1)^2\\&=x^2+y^2+z^2+2(x+y+z)+3\\&=x^2+y^2+z^2+3, \end{aligned}\] tương tự \[\begin{aligned} a^3+b^3+c^3& = x^3+y^3+z^3+3(x^2+y^2+z^2)+3, \\ a^4+b^4+c^4& =x^4+y^4+z^4+4(x^3+y^3+z^3)+6(x^2+y^2+z^2)+3. \end{aligned}\] Bất đẳng thức (2.2.1) trở thành \[3(x^4+y^4+z^4)+6(x^3+y^3+z^3)+x^2+y^2+z^2 \ge 0. \quad (2.2.2)\] Giả sử $xy \ge 0,$ rồi thay $z=-x-y$ vào (2.2.2), ta được \[3[x^4+y^4+(x+y)^4]+6[x^3+y^3-(x+y)^3]+x^2+y^2+(x+y)^2 \ge 0,\] \[3(x^4+2x^3y+3x^2y^2+2xy^3+y^4)+x^2+xy+y^2\ge 9xy(x+y)\] \[3(x^2+xy+y^2)^2+x^2+xy+y^2\ge 9xy(x+y).\] Áp dụng bất đẳng thức AM-GM, ta có \[x^2+xy+y^2 \ge \frac{3}{4}(x+y)^2 \ge 3xy \ge 0, \quad (2.2.3)\] suy ra \[3(x^2+xy+y^2)^2+x^2+xy+y^2\ge 27x^2y^2+\frac{3}{4}(x+y)^2.\] Ta chứng minh \[9x^2y^2+\frac{(x+y)^2}{4} \ge 3xy(x+y).\] Cũng theo bất đẳng thức AM-GM, thì \[9x^2y^2+\frac{(x+y)^2}{4} \ge 2\sqrt{9x^2y^2\cdot\frac{(x+y)^2}{4}}=3xy\left | x+y \right |\ge 3xy(x+y). \quad (2.2.4)\] Đẳng thức xảy ra khi (2.2.3) và (2.2.4) trở thành đẳng thức, tức $x,\,y$ là nghiệm của hệ \[\left\{ \begin{aligned} & x=y \\ & 27x^2y^2 = \frac{3}{4}(x+y)^2 \end{aligned}\right.\] Giải hệ này ta được $x=y=0$ hoặc $x=y=\frac{1}{3},$ suy ra $a=b=c=1,$ hoặc $a=b=\frac{4}{3},\,c=\frac{1}{3}$ cùng các hoán vị. Bài toán được chứng minh.